Tải bản đầy đủ (.pdf) (51 trang)

Bất đẳng thức muirhead và một số vấn đề liên quan

Bạn đang xem bản rút gọn của tài liệu. Xem và tải ngay bản đầy đủ của tài liệu tại đây (382.09 KB, 51 trang )

ĐẠI HỌC THÁI NGUYÊN
TRƯỜNG ĐẠI HỌC KHOA HỌC

BÙI VIỆT LONG

BẤT ĐẲNG THỨC MUIRHEAD VÀ
MỘT SỐ VẤN ĐỀ LIÊN QUAN

LUẬN VĂN THẠC SĨ TOÁN HỌC

THÁI NGUYÊN - 2016


ĐẠI HỌC THÁI NGUYÊN
TRƯỜNG ĐẠI HỌC KHOA HỌC

BÙI VIỆT LONG

BẤT ĐẲNG THỨC MUIRHEAD VÀ
MỘT SỐ VẤN ĐỀ LIÊN QUAN

LUẬN VĂN THẠC SĨ TOÁN HỌC

Chuyên ngành:
Mã số:

PHƯƠNG PHÁP TOÁN SƠ CẤP
60 46 01 13

NGƯỜI HƯỚNG DẪN KHOA HỌC
PGS.TS. HÀ TRẦN PHƯƠNG



THÁI NGUYÊN - 2016


i

Mục lục

Mở đầu

1

Chương 1. Bất đẳng thức Muirhead

3

1.1. Bất đẳng thức Muirhead cho trường hợp bộ hai và ba số . . . . .

3

1.1.1. Một số khái niệm . . . . . . . . . . . . . . . . . . . . . . . .

3

1.1.2. Định lý Muirhead bộ hai và ba số . . . . . . . . . . . . . .

6

1.1.3. Một số ví dụ . . . . . . . . . . . . . . . . . . . . . . . . . . .


9

1.2. Bất đẳng thức Muirhead tổng quát . . . . . . . . . . . . . . . . . . 11
1.2.1. Định lý Muirhead trong trường hợp n biến . . . . . . . . . 11
1.2.2. Bất đẳng thức Muirhead mở rộng . . . . . . . . . . . . . . 15
Chương 2. Một số áp dụng của bất đẳng thức Muirhead

23

2.1. Chứng minh một số bất đẳng thức đại số và hình học . . . . . . . 23
2.1.1. Một số bất đẳng thức đại số . . . . . . . . . . . . . . . . . . 23
2.1.2. Một số bất đẳng thức hình học . . . . . . . . . . . . . . . . 36
2.2. Kết hợp với một số bất đẳng thức khác . . . . . . . . . . . . . . . 40
2.2.1. Một số bất đẳng thức liên quan . . . . . . . . . . . . . . . . 40
2.2.2. Ví dụ áp dụng . . . . . . . . . . . . . . . . . . . . . . . . . . 42
Kết luận

47

Tài liệu tham khảo

48


1

MỞ ĐẦU
Bất đẳng thức là một vấn đề nghiên cứu được hình thành từ khá sớm
của toán học sơ cấp nhưng hiện nay vẫn thu hút được sự quan tâm của
nhiều tác giả. Đây cũng là một phần kiến thức đẹp đẽ, thú vị trong toán

sơ cấp. Do đó các vấn đề về bất đẳng thức luôn cuốn hút được nhiều người
nghiên cứu toán sơ cấp và có nhiều bài tập được sử dụng để thi các kỳ thi
học sinh giỏi quốc gia và quốc tế. Đã có nhiều tác giả trong và ngoài nước
có những nghiên cứu về bất đẳng thức và có nhiều chuyên đề hay, thể hiện
tính thời sự của vấn đề nghiên cứu.
Được hình thành vào đầu thế kỷ XX, bất đẳng thức Muirhead được xuất
hiện trong một công trình nghiên cứu của nhà toán học R. F. Muirhead
vào năm 1903 và là tổng quát hóa khá quan trọng của bất đẳng thức
AM − GM. Nó cho một đánh giá về tổng Symmetric của hai bộ số có
quan hệ ≺ . Có thể nói, bất đẳng thức Muirhead là một công cụ mạnh
trong việc giải một số bài toán về bất đẳng thức có độ phức tạp cao thể
hiện trong việc đã có nhiều bài tập thi học sinh giỏi, Olympic các nước,
khu vực, thế giới - mà việc giải cần dùng đến bất đẳng thức Muirhead.
Hơn nữa, bất đẳng thức Muirhead có thể áp dụng cùng với các bất đẳng
thức khác để xây dựng những bất đẳng thức mới sâu sắc hơn. Mặc dầu
đã có nhiều tác giả quan tâm đến bất đẳng thức Muirhead nhưng việc cải
tiến bất đẳng thức này là khá chậm, hơn một thế kỷ sau (năm 2009) kể
từ công trình của R. F. Muirhead, hai tác giả J. B. Paris và A. Vencovská
mới đưa ra một cải tiến mới về bất đẳng thức này.
Sự lựa chọn đề tài Bất đẳng thức Muirhead và một số vấn đề liên
quan nhằm giới thiệu lại công trình nghiên cứu của R. F. Muirhead và
J. B. Paris và A. Vencovská về đánh giá về tổng Symmetric của hai bộ số


2

thực không âm có quan hệ ≺. Ngoài ra luận văn cũng giới thiệu một số
ví dụ về áp dụng bất đẳng thức Muirhead trong việc chứng minh các bài
tập về bất đẳng thức đã sử dụng trong các kỳ thi học sinh giỏi, Olympic
các nước, khu vực, thế giới.

Luận văn được chia thành hai chương. Chương 1 nhằm giới thiệu các
kiến thức lý thuyết về bất đẳng thức Muirhead và một mở rộng của bất
đẳng thức này. Trong Chương 2 chúng tôi giới thiệu các ví dụ về các bài
toán sử dụng đến bất đẳng thức Muirhead như là một áp dụng của định
lý Muirhead.
Luận văn được thực hiện và hoàn thành tại trường Đại học Khoa học
- Đại học Thái Nguyên. Qua đây tôi xin chân thành cảm ơn các thầy cô
giáo Khoa Toán, Ban Giám hiệu, Phòng Đào tạo nhà trường và các Quý
Thầy Cô giảng dạy lớp Thạc sĩ khóa 8 (6/2014- 6/2016) trường Đại học
Khoa học - Đại học Thái Nguyên đã tận tình truyền đạt những kiến thức
quý báu, đã trang bị kiến thức cơ bản và tạo điều kiện tốt nhất cho tôi
trong quá trình học tập và nghiên cứu.
Tôi xin bày tỏ lòng biết ơn chân thành tới PGS. TS. Hà Trần Phương,
người đã tận tình chỉ bảo, tạo điều kiện và giúp đỡ tôi có thêm nhiều kiến
thức, khả năng nghiên cứu, tổng hợp tài liệu để hoàn thành luận văn một
cách hoàn chỉnh.
Tôi cũng xin gửi lời cảm ơn đến gia đình, bạn bè và các đồng nghiệp đã
động viên, giúp đỡ tôi trong quá trình học tập của mình.
Do thời gian và trình độ còn hạn chế nên luận văn không tránh khỏi
những thiếu sót. Tôi rất mong nhận được sự góp ý của các thầy cô và các
bạn để luận văn được hoàn thiện hơn.
Tôi xin chân thành cảm ơn!
Thái Nguyên, tháng 8 năm 2016
Người viết luận văn

Bùi Việt Long


3


Chương 1

Bất đẳng thức Muirhead
1.1.

Bất đẳng thức Muirhead cho trường hợp bộ hai và ba số

1.1.1.

Một số khái niệm

Định nghĩa 1.1. ([6]) Cho một bộ n số thực không âm a = (a1 , a2 , . . . , an )
và một bộ các số thực dương x = (x1 , x2 , . . . , xn ). Ta định nghĩa
i) Tổng Cyclic (Viết tắt: cyc) của xa11 xa22 . . . xann là đại lượng

xa11 xa22 . . . xann =xa11 xa22 . . . xann + xa21 xa32 . . . xa1n
cyc
n
+ · · · + xan1 xa12 . . . xan−1
.

ii) Tổng Symmetric (Viết tắt: sym) của xa11 xa22 . . . xann là đại lượng

xa11 xa22 . . . xann =

T (a) = T (x; a) =
sym

1
2

n
xaσ(1)
xaσ(2)
. . . xaσ(n)
,

σ∈S(n)

trong đó tổng sym được lấy trên tất cả các hoán vị (σ(1), σ(2), . . . , σ(n))
của (1, 2, . . . , n), S(n) là tập hợp tất cả các hoán vị của {1, 2, ..., n} .
iii) Trung bình Symmetric của xa11 xa22 . . . xann là đại lượng

[x; a] =

1
T (x; a).
n!

Ta có thể sử dụng kí hiệu ngắn gọn [a] thay cho kí hiệu [x; a], T (a) thay
cho T (x; a) khi phần tử x đã được xác định rõ.


4

Ví dụ 1.1. ([2])

ab2 c3 = ab2 c3 + bc2 a3 + ca2 b3 ;
cyc

abc = 6abc.

sym

Ví dụ 1.2. ([4]) Với a = (1, 3, 2) và x = (x1 , x2 , x3 ) thì

T (x; a) = x1 x32 x23 + x1 x33 x22 + x2 x31 x23 + x2 x33 x21 + x3 x31 x22 + x3 x32 x21 .


1
[x; a] = (x1 x32 x23 + x1 x22 x33 + x2 x31 x23 + x2 x33 x21 + x3 x31 x22 + x3 x32 x21 ).
6
Ví dụ 1.3. ([6])

(n − 1)!
1
[(1, 0, 0, ..., 0); (x1 , . . . , xn )] =
(x1 + x2 + ... + xn ) =
n!
n

n

xi
i=1

là trung bình cộng của các số x1 , . . . , xn .


1
1 1
( ; ; ... ); (x1 , . . . , xn ) = n x1 x2 ...xn

n n n
là trung bình nhân của các số x1 , . . . , xn .
Mệnh đề 1.1. ([6])
1. Nếu x1 x2 ...xn = 1 thì

[a1 , a2 , ..., an ] = [(a1 − r), (a2 − r), ..., (an − r)]
đúng với mọi r > 0 sao cho các ai − r
2. Nếu x1 x2 ...xn

0.

1 thì

[a1 , a2 , ..., an ]

[(a1 − r), (a2 − r), ..., (an − r)]

đúng với mọi r > 0 sao cho các ai − r

0.

3. Sử dụng bất đẳng thức AM – GM, với hai bộ số thực không âm a và
b ta có
[a] + [b]
a+b
.
2
2



5

Nhận xét 1.1. Cho bộ các số thực không âm a = (a1 , a2 , . . . , an ) và một
bộ các số thực dương x = (x1 , x2 , . . . , xn ). Nếu b = (aσ(1) , aσ(2) , . . . , aσ(n) ),
trong đó (σ(1), σ(2), . . . , σ(n)) là một hoán vị của {1, 2, . . . , n} thì ta luôn


T (x; a) = T (x; b),

[x; a] = [x; b].

Tiếp theo ta giới thiệu một số khái niệm cơ bản về so sánh các bộ n số.
Cho bộ n số thực không âm a = (a1 , a2 , . . . , an ). Dễ thấy rằng ta luôn có
thể sắp xếp lại trật tự các phần tử trong a để sao cho

a1

a2

···

an .

Do đó trong luận văn này, không mất tính tổng quát ta luôn có thể giả
thiết a1 a2 · · · an khi nói đến bộ n số (a). Ta xem xét khái niệm
về quan hệ ≺ của hai bộ n số thông qua định nghĩa sau.
Định nghĩa 1.2. ([6]) Cho hai bộ n số thực không âm a = (a1 , a2 , . . . , an )
và b = (b1 , b2 , . . . , bn ). Ta nói bộ b trội hơn bộ a, kí hiệu là a ≺ b hay b a
nếu các điều kiện sau thỏa mãn (sau khi sắp xếp lại trật tự các phần tử
trong a, b nếu cần thiết):

1) a1

a2

···

an ; b 1

2) a1 + a2 + · · · + am

b2

···

bn ;

b1 + b2 + · · · + bm với mọi m : 1

m

3) a1 + a2 + · · · + an = b1 + b2 + · · · + bn .
Ví dụ 1.4. ([4])

(2, 1, 0) ≺ (3, 0, 0); (0, 2, 1) ≺ (0, 0, 3),
(4, 0, 0, 0) ≺ (2, 0, 2) vì số phần tử ở hai bộ khác nhau,
(5, 0, −1) ≺ (2, 2, 0) vì có phần tử âm ở một bộ,
(2, 1, 1, 1) ≺ (1, 1, 1, 1)vì 2 + 1 + 1 + 1 = 1 + 1 + 1 + 1,
(4, 1, 1, 1) ≺ (3, 3, 1, 0)vì 4 + 1

3 + 3.


Ví dụ 1.5. ([6])

1
1 1
, ,...,
≺ (1, 0, . . . , 0).
n n
n
n

n

n − 1;


6

1.1.2.

Định lý Muirhead bộ hai và ba số

Định lý 1.2. (Định lý Muirhead bộ hai số, [2]) Cho các số thực dương

a1 , a2 , b1 , b2 thỏa mãn:


 a1 a2 ; b 1 b 2 ;
a1 b1 ;



a1 + a2 = b1 + b2 .
Cho x, y là các số thực dương, khi đó

xb1 y b2 .

xa1 y a2
sym

sym

Đẳng thức xảy ra khi và chỉ khi : a1 = b1 , a2 = b2 hoặc x = y.
Định lý 1.3. (Định lý Muirhead cho bộ ba số, [2]) Cho hai bộ ba số
thực dương a1 , a2 , a3 , b1 , b2 , b3 thỏa mãn:


 a1 a2 a3 ; b1 b2 b3 ;
a1 b1 ; a1 + a2 b1 + b2 ;


a1 + a2 + a3 = b1 + b2 + b3 .
Cho x, y, z là các số thực dương, khi đó

xa1 y a2 z a3
sym

xb1 y b2 z b3 .
sym

Đẳng thức xảy ra khi và chỉ khi : ai = bi ; i = 1, 2, 3 hoặc x = y = z.

Chứng minh. Để chứng minh định lý ta cần đến một bổ đề sau:
Bổ đề 1.4. ([1]) Cho các số thực không âm a1 , a2 , b1 , b2 , thỏa mãn: a1 +

a2 = b1 +b2 ; và max {a1 ; a2 } max {b1 ; b2 } . Khi đó với các số thực dương
x, y , ta có:
xa1 y a2 + xa2 y a1 xb1 y b2 + xb2 y b1 .
Đẳng thức xảy ra khi và chỉ khi a1 = b1 ; a2 = b2 hoặc x = y.
Chứng minh. Không mất tính tổng quát, ta giả sử

a1

a2 , a1

b1 , b1

b2 .


7

Do a1 + a2 = b1 + b2 nên ta có:

xa1 y a2 + xa2 y a1 − xb1 y b2 − xb2 y b1
= xa2 y a2 (xa1 −a2 + y a1 −a2 − xb1 −a2 y b2 −a2 − xb2 −a2 y b1 −a2 )
= xa2 y a2 (xb1 −a2 + y b1 −a2 )(xb2 −a2 − y b2 −a2 )
1
= a a (xb1 + y b1 )(xb2 − y b2 ) 0.
x 2y 2
Bổ đề được chứng minh.
Ta tiếp tục chứng minh định lý. Ta xét hai trường hợp sau:

i) Trường hợp 1. Nếu b1

a1

a2 , điều này kéo theo a1

a1 + a2 − b1 và từ

b1 ta có
max {a1 + a2 − b1 , b1 }

a1
Kéo theo

max {a1 , a2 } = a1

max {a1 + a2 − b1 , b1 } .

Từ

a1 + a2 − b1

b 1 + a3 − b 1 = a3



a1 + a2 − b1

b2


b3

ta có

max{a1 + a2 − b1 , a3 }

max{b2 , b3 }.

Áp dụng Bổ đề 1.4 hai lần ta có:

xa1 y a2 z a3 =
sym

z a3 (xa1 y a2 +xa2 y a1 )
cyc

z a3 (xa1 +a2 −b1 y b1 +xb1 y a1 +a2 −b1 )
cyc

xb1 (y a1 +a2 −b1 z a3 +y a3 z a1 +a2 −b1 )

=
cyc

xb1 (y b2 z b3 +y b3 z b2 )
cyc

xb1 y b2 z b3 .

=

sym


8

ii) Trường hợp 2. Nếu b1

3b1
hơn nữa b1

a2 . Ta có

b1 + b2 + b3 = a1 + a2 + a3

a1 + a2 − b1 và a1

a2

b1

b1 + a2 + a3 ,
a2 + a3 − b1 , nên ta có

max {a2 , a3 } max {b1 , a2 + a3 − b1 }
max {a1 , a2 + a3 − b1 } max {b2 , b3 } .
Áp dụng Bổ đề 1.4 hai lần ta có

xa1 y a2 z a3 =
sym


xa1 (y a2 z a3 +y a3 z a2 )
cyc

xa1 (y b1 z a2 +a3 −b1 +y a2 +a3 −b1 z b1 )
cyc

y b1 (xa1 z a2 +a3 −b1 +xa2 +a3 −b1 z a1 )

=
cyc

y b1 (xb2 z b3 +xb3 z b2 )
cyc

xb1 y b2 z b3 .

=
sym

Dễ dàng thấy rằng nếu a = b hoặc x = y = z thì đẳng thức xảy ra. Định
lý Muirhead cho bộ ba số được chứng minh.
Nhận xét 1.2. Bất đẳng thức Muirhead thường được sử dụng cho trường
hợp ba biến bởi các lý do sau:
- Đối với bất đẳng thức hai biến số, việc áp dụng bất đẳng thức AM −
GM có thể giải quyết dễ dàng và đơn giản hơn so với dùng Định lý Muirhead.
- Đối với bất đẳng thức từ bốn biến trở lên, việc đưa nó về dạng đa thức
đối xứng là một điều tương đối khó khăn, phức tạp.
- Ta có thể sử dụng định lý Muirhead để chứng minh bất đẳng thức
Nesbitt.



9

1.1.3.

Một số ví dụ

Kỹ thuật chung để sử dụng định lý Muirhead trong chứng minh bất
đẳng thức ta cần thực hiện như sau:
Bước 1: Phân tích
- Biến đổi bất đẳng thức cần chứng minh về dạng tổng các đa thức đối
xứng ở cả hai vế của bất đẳng thức;
- Biểu diễn các đa thức đối xứng theo ký hiệu quy ước trong lý thuyết
nói trên.
Bước 2: Đánh giá
- Làm mất dần các đa thức đối xứng có giá trị lớn ở vế có giá trị nhỏ
hơn; thay vào đó là các đa thức nhỏ nhất có thể.
- Phép đánh giá được thực hiện nhờ các nguyên tắc sau:
+ Sử dụng đánh giá các bất đẳng thức sẵn có;
+ Tìm tòi, dự đoán các bất đẳng thức nhỏ hơn cần chứng minh;
+ Sử dụng các cách biến đổi để tạo ra cách đánh giá mới.
Bây giờ ta xét một số ví dụ cụ thể.
Ví dụ 1.6. ([1]) Với a, b, c > 0, α ≥ 1 thỏa mãn abc = 1 thì




+
+
b+c c+a a+b


3
.
2

Thật vậy, bất đẳng thức cần chứng minh tương đương với:
α−1
3
(a + b)(b + c)(c + a)(abc) 3 .
2

aα (a + b)(a + c)
cyc

Bất đẳng thức này tương đương với

(aα+2 +aα+1 b + aα+1 c + aα bc)

2
cyc

3(

a
sym

α+5
3

b


α+2
3

c

α−1
3

+ 2a

α+2
3

b

α+2
3

c

α+2
3

)


10

Tương đương với


aα+2 −
sym

a

b

α+2
3

c

α−1
3

sym

aα+1 b −

+2

a

aα bc −

+

α+5
3


b

α+2
3

c

α−1
3

sym

sym

a

α+2
3

b

α+2
3

c

α+2
3


0.

sym

sym

Do α

α+5
3

1 nên
(α + 2, 0, 0)

α+5 α+2 α−1
,
,
);
3
3
3
α+2 α+2 α+2
(
,
,
).
3
3
3


(α + 1, 1, 0)

(α, 1, 1)

(

Nên bất đẳng thức cuối cùng đúng theo bất đẳng thức Muirhead trong
trường hợp bộ ba số. Dễ dàng kiểm tra được đẳng thức xảy ra khi và chỉ
khi a = b = c = 1.
1
1
1
Nhận xét. Bài toán trên nếu ta chọn α = 2 và đặt a = , b = , c =
x
y
z
thì ta sẽ thu được bài toán IMO 1995 - Ví dụ 2.3, trang 25.
Ví dụ 1.7. ([1]) Cho ba số thực dương a, b, c > 0 khi đó ta có bất đẳng
thức:

a3
b3
c3
+
+
b2 − bc + c2 c2 − ca + a2 a2 − ab + b2

3(ab + bc + ca)
.
a+b+c


Thật vậy, vì

a+b+c

3(ab + bc + ca)
a+b+c

nên để chứng minh bài toán, ta sẽ chứng minh khẳng định sau:

a3
b3
c3
+
+
b2 − bc + c2 c2 − ca + a2 a2 − ab + b2
Bất đẳng thức trên tương đương với

cyc

a3 (b + c)
b3 + c3

a + b + c.

a + b + c.


11


Hay

a3 (b + c)(a3 + b3 )(a3 + c3 )

(a + b + c)(a3 + b3 )(b3 + c3 )(c3 + a3 ).

cyc

Tương đương với bất đẳng thức

(a3 b + a3 c)(a6 + a3 b3 + b3 c3 + c3 a3 )

a6 b3 + 2a3 b3 c3

(a + b + c)

cyc

sym

hay

sym

sym

a4 b3 c3

a6 b 3 c +


a6 b4 +

a9 b +

sym

sym

sym

sym

a4 b 3 c 3 .

a6 b 3 c +

a6 b 4 +

a7 b3 +

sym

sym

Rút gọn bất đẳng thức trên ta được

a9 b
sym

a7 b 3 .

sym

Bất đẳng thức cuối cùng đúng theo bất đẳng thức Muirhead. Đẳng thức
xảy ra khi và chỉ khi a = b = c.

1.2.

Bất đẳng thức Muirhead tổng quát

1.2.1.

Định lý Muirhead trong trường hợp n biến

Cho bộ n các số thực không âm a = (a1 , a2 , . . . , an ) và x = (x1 , x2 , . . . , xn )
là một bộ các biến thực dương. Ta nhắc lại tổng sym các phần tử của x
với bộ số mũ a là đại lượng
1
2
n
xaσ(1)
xaσ(2)
. . . xaσ(n)
,

T (x; a) =
σ∈S(n)

trong đó tổng sym được lấy trên tất cả các hoán vị của {1, 2, . . . , n}, S(n)
là tập hợp tất cả các hoán vị của {1, 2, ..., n} . Ta có định lý sau được gọi
là định lý Muirhead.

Định lý 1.5. ([10]) Cho hai bộ n các số thực không âm a = (a1 , a2 , . . . , an )
và b = (b1 , b2 , . . . , bn ). Khi đó, tổng sym T (x; a) so sánh được với tổng sym


12

T (x; b) đối với mọi bộ các biến thực dương x = (x1 , x2 , . . . , xn ) khi và chỉ
khi a và b so sánh được với nhau theo quan hệ ≺. Nếu a ≺ b thì
T (x; a)

T (x; b).

(1.1)

Đẳng thức xảy ra khi và chỉ khi hoặc a ≡ b hoặc x1 = x2 = · · · = xn .
Chứng minh. Trước hết ta chứng minh điều kiện cần. Giả sử có

T (x; a)

T (x; b).

(1.2)

đúng với mọi bộ các biến thực (x1 , x2 , . . . , xn ). Ta lấy

x1 = x2 = · · · = xn = c,
trong đó c là hằng số, bất đẳng thức (1.2) cho ta

c


ai

c

bi

(1.3)

đúng với mọi giá trị c > 0 lớn tùy ý hay nhỏ tùy ý. Điều này kéo theo

a1 + a2 + · · · + an = b1 + b2 + · · · + bn .
Tiếp theo, cho x1 = c, x2 = · · · = xn = 1, bất đẳng thức (1.2) cho ta

ca1 + ca2 + · · · + can

cb1 + cb2 + · · · + cbn

(1.4)

đúng với mọi giá trị c > 0 tùy ý. Chú ý rằng do

a1

a2

···

an ,

b1


b2

···

bn

nên ca1 + ca2 + · · · + can là đa thức bậc a1 và cb1 + cb2 + · · · + cbn là đa thức
bậc b1 đối với cùng biến c. Do bất đẳng thức (1.4) đúng với mọi c > 0
lớn tùy ý nên suy ra a1 b1 . Tương tự, với mỗi k = 1, . . . , n − 1, ta cho

x1 = x2 = · · · = xk = c và xk+1 = · · · = xn = 1, bất đẳng thức (1.2) cho
ta
P (c)

Q(c)

(1.5)

đúng với mọi giá trị c > 0 tùy ý, trong đó P (c) là đa thức bậc a1 + a2 +

· · · + ak , Q(c) là đa thức bậc b1 + b2 + · · · + ak . Do (1.5) đúng với mọi giá
trị c > 0 lớn tùy ý nên ta có
a1 + a2 + · · · + ak

b1 + b2 + · · · + ak .


13


Như vậy

a1 + a2 + · · · + ak

b1 + b2 + · · · + ak

đúng với mỗi k = 1, . . . , n − 1.
Bây giờ ta chứng minh điều kiện đủ. Trước hết ta xây dựng một toán
tử tuyến tính L để biến đổi một bộ n số. Ta gọi β = (β1 , β2 , . . . , βn ) là
một bộ n số nguyên không âm. Gọi βk và βl là hai phần tử phân biệt của

β thỏa mãn βk > βl , ta viết
βk = ρ + τ,
Khi đó với một số σ : 0

βk = ρ − τ

(0 < τ

ρ).

σ<τ

ρ ta xác định bộ α như sau:
τ −σ
τ +σ
βk +
βl ,
αk = ρ + σ =



τ +σ
τ −σ
βk +
βl ,
αl = ρ − σ =


αν = βν (ν = k, ν = l).

Khi đó ta viết α = L(β). Chú ý rằng, cách xác định toán tử L như trên
không yêu cầu một trong hai dãy α, β là dãy giảm. Để chứng minh định
lý ta cần hai bổ đề sau:
Bổ đề 1.6. ([10]) Nếu a = L(b) thì T (x; a)

T (x; b). Dấu bằng xảy ra

khi x1 = x2 = · · · = xn .
Chứng minh. Ta sắp xếp lại (nếu cần thiết) thứ tự các phần tử trong bộ

b sao cho k = 1, l = 2, tức là b1 = b2 . Khi đó
T (x; b) − T (x; a)
ρ−τ
ρ+τ
ρ−σ
ρ+σ
xb33 . . . xbnn (xρ+τ
+ xρ−τ
− xρ+σ
− xρ−σ

1 x2
1 x2
1 x2
1 x2 )

=
sym

(x1 x2 )ρ−τ xb33 . . . xbnn (xτ1 +σ − xτ2 +σ )(xτ1 −σ − xτ2 −σ )

=
sym

Hiển nhiên, nếu x1 = x2 = · · · = xn thì biến đổi trên cho ta

T (x; b) − T (x; a) = 0.
Bổ đề được chứng minh.

0.


14

Bổ đề 1.7. ([10]) Nếu a ≺ b và a = b thì a có thể nhận được từ b bằng
cách áp dụng một số hữu hạn lần phép biết đổi L. Tức là tồn tại một số
nguyên dương m sao cho a = Lm (b).
Chứng minh. Gọi m là số các chỉ số ν ∈ {1, 2, . . . , n} sao cho bν −aν = 0,
hiển nhiên m là một số nguyên dương. Ta sẽ chứng minh ta có thể áp dụng
phép biến đổi L m lần để nhận được a từ b. Thật vậy, từ điều kiện


a1 + a2 + · · · + an = b 1 + b 2 + · · · + b n
ta suy ra trong m chỉ số ν làm cho bν − aν = 0 thì sẽ có cả chỉ số ν làm
cho bν − aν > 0 và có cả chỉ số ν làm cho bν − aν < 0 nhưng chỉ số ν
đầu tiên trong chúng sẽ làm cho bν − aν > 0. Ta sẽ chọn k, l như sau: l là
chỉ số nhỏ nhất làm cho bl − al < 0 và k < l là chỉ số lớn nhất làm cho
bk − ak > 0. Hiển nhiên bk > ak al > bl nên bk > bl . Đặt

bk = ρ + τ ;

bl = ρ − τ



σ = max{|ak − ρ|, |al − ρ|}.
Khi đó ít nhất một trong hai đẳng thức sau đúng:

al − ρ = −σ;
vì ak

ak − ρ = σ,

al . Hơn nữa ta cũng có σ < τ vì ak < bk và al > bl . Đặt
ck = ρ + σ,
cν = bν

cl = ρ − σ,

(ν = k, ν = l).

Khi đó hoặc ck = ak hoặc al = cl .

Hiển nhiên c = L(b) và số chỉ số ν ∈ {1, 2, . . . , n} sao cho cν − aν = 0
bằng m − 1. Đặt c = (c1 , c2 , . . . , cn ), ta dễ dàng kiểm tra dãy c là dãy giảm
và có quan hệ ≺ với dãy a, tức là a ≺ c. Hơn nữa do σ < τ nên c ≺ b.
Ta lặp lại quá trình trên khi thay thế dãy b bởi dãy c và thực hiện m
lần như thế ta sẽ nhận được dãy a.
Ta tiếp tục chứng minh định lý. Từ Bổ đề 1.6 ta suy ra tồn tại m sao
cho a = Lm (b). Từ Bổ đề 1.7 ta suy ra T (x; a)
T (x; b). Định lý được
chứng minh.


15

Nhận xét 1.3. ([8]) Bất đẳng thức AM − GM là một trường hợp đặc
biệt của bất đẳng thức Muirhead. Thật vậy, với n số thực không âm bất
kỳ: a1 , a2 , .., an ; (1 < n ∈ N), kí hiệu
a1 + a2 + .. + an
AM =
n


GM =


n

a1 .a2 . . . an

Kí hiệu a = (a1 , a2 , .., an ). Khi đó


1
T a; (1, 0, . . . , 0)
n!

AM =


1
1
1 1
T a; , , . . . ,
n!
n n
n

GM =

.

Hiển nhiên

1 1
1
, ,...,
≺ (1, 0, . . . , 0)
n n
n
nên từ Định lý 1.5 ta suy ra bất đẳng thức quen thuộc là AM
1
1 1

, ,...,
= (1, 0, . . . , 0)
n n
n
nên đẳng thức xẩy ra khi và chỉ khi a1 = a2 = · · · = an .
1.2.2.

GM. Do

Bất đẳng thức Muirhead mở rộng

Ta nói một bộ gồm r tập hợp {S1 , . . . , Sr } là một phép phân hoạch của
tập {1, 2, . . . , k} nếu Si ∩ Sj = ∅ và ∪rj=1 Sj = {1, 2, . . . , k}.
Cho tập chỉ số S ⊂ {1, 2, . . . , k}, số nguyên m > 0 và các số thực
dương p1 , p2 , . . . , pk , tổng

m

pi
i∈S

pi

biểu diễn lũy thừa của

m

được hiểu là tổng các số hạng trong

mà ta chỉ tính các số hạng có bội khác 0


i∈S

đối với tất cả các pj với j ∈ S.
Hiển nhiên nếu m < |S| thì

pi

m

= 0. Nếu S = ∅ ta quy ước

i∈S

pi
i∈S

m


1
=
0

nếu m = 0
nếu m > 0.


16


Bây giờ ta sẽ chứng minh một mở rộng của bất đẳng thức Muirhead.
Định lý 1.8. ([7]) Cho (n1 , n2 , . . . , nr ) , (m1 , m2 , . . . , mr ) là hai bộ r các
số thực giảm, không âm sao cho (m1 , m2 , . . . , mr ) ≺ (n1 , n2 , . . . , nr ) và
p1 , p2 , . . . , pk là các số thực dương. Khi đó
r

r

nj

mj

pi
par j=1

pi
par j=1

i∈Sj

(1.6)

i∈Sj

lấy trên tất cả các phép phân hoạch {S1 , . . . , Sr } của tập

trong đó
par

{1, 2, . . . , k}.

Chứng minh. Để chứng minh định lý ta cần bổ đề chìa khóa sau
Bổ đề 1.9. ([7]) Cho m, n, k ∈ N, n > m

0, X = {1, 2, . . . , k},

p1 , p2 , . . . , pk ∈ R là các số thực dương. Đặt
n

m

pi

P (n, m) =
Q⊂X

pi

i∈Q

.

i∈X\Q

Khi đó

P (n + 1, m)

P (n, m + 1).

(1.7)


Chứng minh. Ta thấy các trường hợp k < 2 và k > n + m + 1 là đơn
giản. Ta chứng minh các trường hợp còn lại. Ta xét các số hạng dạng

pi1 pi2 . . . pin+m+1 trong biểu diễn đầy đủ P (n + 1, m), trong đó
(n+1)

pi1 pi2 . . . pin+1 ∈

,

pi
i∈Q

m

pin+2 pi2 . . . pin+m+1 ∈

pi
i∈X\Q

với mỗi tập con Q ⊂ X , trong đó biểu diễn đầy đủ P (n + 1, m) được
hiểu là tổng các tích của các số pi trong việc nhân
i∈X\Q pi

i∈Q pi

n lần với

m lần và giữ các pi theo đúng thứ tự trong quá trình nhân,


(không gộp thành lũy thừa nếu các pi giống nhau) và bỏ đi các tích mà


17

không chứa các pi ít nhất một lần. Ta có thể chia thành hai trường hợp
sau:
(a1) in+1 ∈ {i1 , i2 , . . . , in },
(a2 G, T ) in+1 ∈ {i1 , i2 , . . . , in }, đặt

G = {im+1 , im+2 , . . . , in }\{i1 , i2 , . . . , im }
T = {j : m < j

n, ij ∈ G},

khi đó G ⊂ X, T ⊂ {m + 1, m + 2, . . . , n}.
Tương tự, ta xem xét số hạng pi1 pi2 . . . pin+m+1 trong biểu diễn đầy đủ
P (n, m + 1), trong đó
n

pi1 pi2 . . . pin ∈

pi

,

i∈Q
(m+1)


pin+1 pi2 . . . pin+m+1 ∈

pi

,

i∈X\Q

với mỗi tập con Q ⊂ X , ta cũng có thể chia thành hai trường hợp sau:

(b1) in+1 ∈ {in+2 , in+3 , . . . , in+m+1 },
(b2 G, T ) in+1 ∈ {in+2 , in+3 , . . . , in+m+1 }, đặt
G = {im+1 , im+2 , . . . , in }\{i1 , i2 , . . . , im }
T = {j : m < j

n, ij ∈ G},

khi đó G ⊂ X, T ⊂ {m + 1, m + 2, . . . , n}.
Bây giờ ta kết hợp P (n + 1, m) và P (n, m + 1) theo phép phân chia
như trên. Các số hạng thuộc trường hợp (a1 ) hoặc (b1 ) hiển nhiên có thể
bỏ qua trong việc xem xét vì những số hạng như thế sẽ xuất hiện trong cả

P (n+1, m) và P (n, m+1) với những tập Q thích hợp (chú ý rằng P (n, m)
được lấy tổng qua tất cả các tập con Q của X nên sẽ có hai tập con Q và
Q của X để số hạng này đều có trong P (n + 1, m) và P (n, m + 1)).
Với mỗi bộ tập hợp G, T cố định, từ trường hợp (a2 G, T ) vào


18


P (n + 1, m) là
|T |

m

pi

pi

i∈Q

Q⊂X\G

n−m−|T |

pi

i∈G

i∈Q
m

×

pi

pi

i∈Q∪G


,

(1.8)

i∈X\(Q∪G)

trong khi đó từ trường hợp (b2 G, T ) vào P (n, m + 1) là
|T |

m

pi

pi

i∈Q

Q⊂X\G

n−m−|T |

pi

i∈G

i∈Q
m

×


pi

pi

i∈X\(G∪Q)

.

(1.9)

i∈X\(G∪Q)

Lấy (1.8) trừ (1.9) ta được phần chênh lệch bao gồm
|T |

m

pi

pi

i∈Q

Q⊂X\G

n−m−|T |

pi

i∈G


i∈Q
m

×

pi

pi

i∈G

(1.10)

i∈X\(Q∪G)


|T |

m

pi
Q⊂X\G

m

pi

i∈Q


pi

i∈G

i∈X\(Q∪G)
n−m−|T |

n+1−m−|T |

×



pi

pi

pi

i∈Q

i∈Q

.

(1.11)

i∈X\(Q∪G)

Dễ thấy (1.10) là một đại lượng không âm. Bây giờ ta xét (1.11). Ta thấy

việc lấy tổng trên tất cả các tập con Q ⊂ X\G trong (1.11) tương tự như
lấy tổng trên phần bù Q của Q trong X\G, nên (1.11) tương đương với
|T |

m

pi
Q⊂X\G

i∈X\(G∪Q )

m

pi
i∈G

pi
i∈Q

n+1−m−|T |

×

pi
i∈X\(G∪Q )

n−m−|T |




pi
i∈X\(G∪Q )

pi

.

i∈Q

(1.12)


19

Ta viết lại Q trong (1.12) thành Q thì ta thấy tổng (1.11) và (1.12) là
|T |

m

pi
Q⊂X\G

m

pi

i∈Q

pi


i∈G

i∈X\(Q∪G)

× An+1−m−|T | − An−m−|T | B + B n+1−m−|T | − B n−m−|T | A ,
trong đó A =

i∈Q pi , B

=

i∈X\(Q∪G) pi ,

là các đại lượng không âm vì

(An+1−m−|T | − An−m−|T | B + B n+1−m−|T | − B n−m−|T | A)
= (An−m−|T | − B n−m−|T | )(A − B)

0.

Điều này kéo theo bất đẳng thức (1.7).
Từ Bổ đề (1.9) ta có
Hệ quả 1.10. ([7]) Cho (n1 , n2 , . . . , nr )

(m1 , m2 , . . . , mr ) là hai bộ r
các số thực giảm, không âm và 1
ir. Giả sử rằng nl = ml với
mọi l : 1 l r, l = i, j, và mi = ni − 1 mj = nj + 1. Khi đó với mỗi
bộ p1 , p2 , . . . , pk các số thực dương ta có:

r

nj

r

mj

pi
par j=1

pi
par j=1

i∈Sj

i∈Sj

lấy qua tất cả các phép phân hoạch {S1 , . . . , Sr } của tập

trong đó
par

{1, 2, . . . , k}.
Chứng minh. Ta cố định các tập Sl với l = i, j và đặt X = Si ∪ Sj ,
Q = Si . Sử dụng Bổ đề (1.9) ta sẽ có kết luận của hệ quả.
Bây giờ ta chứng minh định lý. Bằng cách quy nạp ta cũng chứng minh
được kết luận của Hệ quả 1.10 vẫn còn đúng khi mi = ni −k mj = nj +k.
Điều này kéo theo nếu


(m1 , m2 , . . . , mr ) = L(n1 , n2 , . . . , nr )
thì
r

nj

r

mj

pi
par j=1

i∈Sj

pi
par j=1

i∈Sj

,

(1.13)


20

trong đó L là phép biến đổi tuyến tính được đề cập đến đến trong Bổ đề 1.7.
Bằng quy nạp ta cũng chứng minh được nếu


(m1 , m2 , . . . , mr ) = Lm (n1 , n2 , . . . , nr ),
trong đó m là một số nguyên dương nào đó thì (1.13) vẫn còn đúng.
Bây giờ ta xét hai bộ (m1 , m2 , . . . , mr ), (n1 , n2 , . . . , nr ) trong giả thiết
của định lý, theo Bổ đề 1.7, tồn tại một số hữu hạn m để sao cho

(m1 , m2 , . . . , mr ) = Lm (n1 , n2 , . . . , nr ).
Sử dụng công thức (1.13) ta sẽ có kết luận của định lý.
Nhận xét 1.4. ([7])
1. Dựa vào chứng minh của Định lý 1.8, dấu đẳng thức trong bất đẳng
thức (1.6) khi và chỉ khi (m1 , m2 , . . . , mr ) = (n1 , n2 , . . . , nr ) hoặc p1 =
p2 = · · · = pk .
2. Nếu r = k thì Định lý 1.8 sẽ nhận lại Định lý 1.5. Vì trong trường hợp
này chỉ có phép phân hoạch mà mỗi tập Sj có đúng một phần tử mới từ
nj

các tổng
par

k
j=1

i∈Sj

pi

nj

và lúc này
par


k
j=1

i∈Sj

pi

chính

là tổng sym của các phần tử p1 , p2 , . . . , pk ứng với bộ số mũ n1 , n2 , . . . , nk .
Từ Định lý 1.8 ta có hệ quả:
Hệ quả 1.11. ([7]) Cho 0 < p1 , . . . , pk ∈ R và n
bất đẳng thức
n
k−|V ∪T |

(−2)

pi −

pi

i∈V

0. Khi đó ta có

m

pi


V,T ⊂{1,2,...,k}
V ∩T =∅

m

i∈T

i∈V

pi

0.

i∈T

Chứng minh. Bằng cách quy nạp theo |Q| ta có thể chỉ ra rằng với mỗi
tập con Q ⊂ X = {1, 2, . . . , k} ta có
n

pi
i∈Q

n

=

|Q|−|V |

(−1)
V ⊂Q


pi
i∈V

.

(1.14)


21

Hiển nhiên điều này cũng đúng khi Q = ∅ và nếu điều này đúng với mọi
tập con thực sự của Q thì
n

pi

n

=



pi

i∈Q

n

i∈Q


pi
Z⊂Q

i∈Z

n

=

n
|Z|−|V |



pi

(−1)
Z⊂Q V ⊂Z

i∈Q

pi
i∈V

n

=

n

|Z|−|V |



pi

(−1)

pi

V ⊂Q V ⊂Z⊂Q

i∈Q

i∈V
n

n

=

pi

(−1)|Q|−|V |

+

pi

V ⊂Q


i∈Q

i∈V
n

(−1)|Q|−|V |

=

V ⊂Q

.

pi

(1.15)

i∈V

vì V ⊂ Q nên

(−1)|Z|−|V | = (−1)|Q|−|V | +

0=

V ⊂Z⊂Q

(−1)|Z|−|V | .
V ⊂Z⊂Q


Điều này kéo theo
n

m

pi
Q⊂X

pj

i∈Q

j∈X\Q
n

pi

=
Q⊂X

V ⊂Q

m

(−1)

i∈V

i∈V


i∈T

m

pi
V,T ⊂X
V ∩T =∅

(−1)|Q|−|V |+k−|Q|−|T |

pj
j∈T

V ⊂Q
V ∩T ⊂X\Q
m

n
k−|V |−|T |

=

2
V,T ⊂X
V ∩T =∅

(−1)k−|Q|−|T |

pi

T ⊂X\Q

n

=

|Q|−|V |

k−|V |−|T |

(−1)

pi
i∈V

Điều này kéo theo bất đẳng thức P (n + 1, m)

pj

.

j∈T

P (n, m + 1) là tương


22

đương với
n+1

k−|V |−|T |

2
V,T ⊂X
V ∩T =∅

m

pi

pj

i∈V

j∈T
n
k−|V |−|T |

2
V,T ⊂X
V ∩T =∅

và điều này sẽ kéo theo kết luận của hệ quả.

m+1

pi
i∈V

pj

j∈T


×